Last visit was: 25 Apr 2024, 04:50 It is currently 25 Apr 2024, 04:50

Close
GMAT Club Daily Prep
Thank you for using the timer - this advanced tool can estimate your performance and suggest more practice questions. We have subscribed you to Daily Prep Questions via email.

Customized
for You

we will pick new questions that match your level based on your Timer History

Track
Your Progress

every week, we’ll send you an estimated GMAT score based on your performance

Practice
Pays

we will pick new questions that match your level based on your Timer History
Not interested in getting valuable practice questions and articles delivered to your email? No problem, unsubscribe here.
Close
Request Expert Reply
Confirm Cancel
SORT BY:
Date
Tags:
Poor Qualityx      
Show Tags
Hide Tags
User avatar
Manager
Manager
Joined: 07 Mar 2016
Posts: 54
Own Kudos [?]: 167 [12]
Given Kudos: 163
Send PM
User avatar
Manager
Manager
Joined: 07 Mar 2016
Posts: 54
Own Kudos [?]: 167 [1]
Given Kudos: 163
Send PM
Manager
Manager
Joined: 27 Jan 2013
Posts: 84
Own Kudos [?]: 474 [0]
Given Kudos: 41
Location: India
GMAT 1: 700 Q49 V35
GPA: 3.7
Send PM
CR Moderator
Joined: 14 Dec 2013
Posts: 2413
Own Kudos [?]: 15266 [1]
Given Kudos: 26
Location: Germany
Schools:
GMAT 1: 780 Q50 V47
WE:Corporate Finance (Pharmaceuticals and Biotech)
Send PM
Re: A recent report determined that [#permalink]
1
Kudos
Expert Reply
RaghavSingla wrote:
The assumption and conclusion seem to be same. IMO answer should be A.


This question seems to be flawed. Could you explain (with a numerical example) why you feel that A is an assumption?
Manager
Manager
Joined: 03 Jul 2016
Posts: 51
Own Kudos [?]: 46 [0]
Given Kudos: 166
Send PM
Re: A recent report determined that [#permalink]
What is the source of this question??
Current Student
Joined: 18 Jun 2016
Posts: 221
Own Kudos [?]: 613 [0]
Given Kudos: 111
Location: United States (NY)
GMAT 1: 720 Q50 V38
GMAT 2: 750 Q49 V42
GPA: 4
WE:General Management (Other)
Send PM
Re: A recent report determined that [#permalink]
sayantanc2k wrote:
RaghavSingla wrote:
The assumption and conclusion seem to be same. IMO answer should be A.


This question seems to be flawed. Could you explain (with a numerical example) why you feel that A is an assumption?

I agree with you. This question is Flawed and as far as A is concerned, it is going against the stated premise that even though only 8% of the drivers hold yearly permits they represent 15% of all cars entering the park.
Intern
Intern
Joined: 05 Jun 2016
Posts: 15
Own Kudos [?]: 4 [0]
Given Kudos: 772
Send PM
Re: A recent report determined that [#permalink]
Choice D just repeats the conclusion. How can you assume what you set out to prove? None of the other answer choices make any sense either. Source please?
Senior Manager
Senior Manager
Joined: 05 Sep 2016
Status:DONE!
Posts: 274
Own Kudos [?]: 101 [0]
Given Kudos: 283
Send PM
Re: A recent report determined that [#permalink]
I was able to get answer choices down to C and D. I did end up choosing the correct answer, but would like some clarification as to why C is incorrect.

Can someone please help me out with this?

Thanks in advance :)
CR Moderator
Joined: 14 Dec 2013
Posts: 2413
Own Kudos [?]: 15266 [0]
Given Kudos: 26
Location: Germany
Schools:
GMAT 1: 780 Q50 V47
WE:Corporate Finance (Pharmaceuticals and Biotech)
Send PM
Re: A recent report determined that [#permalink]
Expert Reply
lawiniecke wrote:
I was able to get answer choices down to C and D. I did end up choosing the correct answer, but would like some clarification as to why C is incorrect.

Can someone please help me out with this?

Thanks in advance :)


This question is flawed. Option D cannot be an assumption because it is the main conclusion of the passage (even having almost the same wording).

Option C is also not a required assumption. The passage states that out of every 100 drivers 8 drivers had yearly permit. There may be other drivers who have yearly permit, but never entered the park. Whether there are any such drivers is out of the context of this argument.
Board of Directors
Joined: 17 Jul 2014
Posts: 2163
Own Kudos [?]: 1180 [0]
Given Kudos: 236
Location: United States (IL)
Concentration: Finance, Economics
GMAT 1: 650 Q49 V30
GPA: 3.92
WE:General Management (Transportation)
Send PM
Re: A recent report determined that [#permalink]
I'm closing the topic by request.
I narrowed down to D as well, but I don't think this question is good for practice, as it adds no value to people who prepare for GMAT.

This Question is Locked Due to Poor Quality
Hi there,
The question you've reached has been archived due to not meeting our community quality standards. No more replies are possible here.
Looking for better-quality questions? Check out the 'Similar Questions' block below for a list of similar but high-quality questions.
Want to join other relevant Problem Solving discussions? Visit our Critical Reasoning (CR) Forum for the most recent and top-quality discussions.
Thank you for understanding, and happy exploring!
GMAT Club Bot
Re: A recent report determined that [#permalink]
Moderators:
GMAT Club Verbal Expert
6920 posts
GMAT Club Verbal Expert
238 posts
CR Forum Moderator
832 posts

Powered by phpBB © phpBB Group | Emoji artwork provided by EmojiOne